Inclined Plane Problem :D (1 Viewer)

seventhroot

gg no re
Joined
Aug 3, 2014
Messages
2,809
Gender
Male
HSC
2013
From Fundamentals of Physics 10th Ed

When I resolve the forces I get this:


but the answers have this:


I can't see how the forces in the y direction is meant to be FN - mgcosθ = 0

I think the error stems the mg component but I can't see where I am going wrong. inb4can'tdoyear9trigonometry

I know the mgsinθ part is a mistake; it should be mgtanθ

The answers are not incorrect (apparently)

will rep :D
 

glittergal96

Active Member
Joined
Jul 25, 2014
Messages
418
Gender
Female
HSC
2014
From Fundamentals of Physics 10th Ed

When I resolve the forces I get this:


but the answers have this:


I can't see how the forces in the y direction is meant to be FN - mgcosθ = 0

I think the error stems the mg component but I can't see where I am going wrong. inb4can'tdoyear9trigonometry

I know the mgsinθ part is a mistake; it should be mgtanθ

The answers are not incorrect (apparently)

will rep :D
Why do you think the part is a mistake? It looks right to me.

Read the paragraph carefully, we are rotating coordinate axes so x is "up the slope" and y is the upwards normal to the slope.

The weight-force contributes in THIS x-direction and in THIS y-direction.

(When resolving a 2-d force vector into two perpendicular components using a triangle, remember that the original force needs to be the hypotenuse of this triangle!)
 

seventhroot

gg no re
Joined
Aug 3, 2014
Messages
2,809
Gender
Male
HSC
2013
Why do you think the part is a mistake? It looks right to me.

Read the paragraph carefully, we are rotating coordinate axes so x is "up the slope" and y is the upwards normal to the slope.

The weight-force contributes in THIS x-direction and in THIS y-direction.
what I did was rotate the problem so that the the Fn axis was the "y axis" and Tension was the "x axis" so that I don't have to worry about splitting the other 2

I thought it was wrong because

(When resolving a 2-d force vector into two perpendicular components using a triangle, remember that the original force needs to be the hypotenuse of this triangle!)
why is this? I think this is where I am going wrong

inb4n00b question; how did you break it up into components?
 

glittergal96

Active Member
Joined
Jul 25, 2014
Messages
418
Gender
Female
HSC
2014
I don't get your logic, anyway yeah what the answers do is right.

why is this? I think this is where I am going wrong

inb4n00b question; how did you break it up into components?
Because we are resolving into perpendicular components, so the other two sides need to be perpendicular. Also, it makes sense that each component should be less that the total force in magnitude.

To break it up into components I drew a triangle with hypotenuse the weight force and other two sides parallel and perpendicular to the incline and used basic trig.
 

seventhroot

gg no re
Joined
Aug 3, 2014
Messages
2,809
Gender
Male
HSC
2013
I don't get your logic, anyway yeah what the answers do is right.

Because we are resolving into perpendicular components, so the other two sides need to be perpendicular. Also, it makes sense that each component should be less that the total force in magnitude.

To break it up into components I drew a triangle with hypotenuse the weight force and other two sides parallel and perpendicular to the incline and used basic trig.
so what I did was make the x and y axis in the T and F directions respectively and with my "basic trig" was this:



and it looks right? but that's still not what they have

the hypotenuse is y so would the magnitude of that be greater than mg?
 

aDimitri

i'm the cook
Joined
Aug 22, 2013
Messages
505
Location
Blue Mountains
Gender
Male
HSC
2014
so what I did was make the x and y axis in the T and F directions respectively and with my "basic trig" was this:



and it looks right? but that's still not what they have

the hypotenuse is y so would the magnitude of that be greater than mg?
how can the magnitude of Y be greater than mg when it is a directional component of mg?
mg is the hypotenuse, the line along which you drew x should be perpendicular to T if you are going to resolve along those directions
 

glittergal96

Active Member
Joined
Jul 25, 2014
Messages
418
Gender
Female
HSC
2014
so what I did was make the x and y axis in the T and F directions respectively and with my "basic trig" was this:



and it looks right? but that's still not what they have

the hypotenuse is y so would the magnitude of that be greater than mg?
reread what I said about the two non weight-force sides of the triangle.

also, see what adimitri says above.
 

SquareHeartsAdrita

cya later alligator
Joined
Aug 3, 2014
Messages
209
Gender
Male
HSC
2013
I see where I am going wrong now



I was drawing the triangle wrong lel

thanks to you all; I get what you were all saying now. I "liked" and repped you both :D
 

Users Who Are Viewing This Thread (Users: 0, Guests: 1)

Top